diff --git a/Analisi I/Parte teorica/2023-03-30, Es. applicazione dei teoremi sulla continuità/main.pdf b/Analisi I/Parte teorica/2023-03-30, Es. applicazione dei teoremi sulla continuità/main.pdf index da679de..e2845d7 100644 Binary files a/Analisi I/Parte teorica/2023-03-30, Es. applicazione dei teoremi sulla continuità/main.pdf and b/Analisi I/Parte teorica/2023-03-30, Es. applicazione dei teoremi sulla continuità/main.pdf differ diff --git a/Analisi I/Parte teorica/2023-03-30, Es. applicazione dei teoremi sulla continuità/main.tex b/Analisi I/Parte teorica/2023-03-30, Es. applicazione dei teoremi sulla continuità/main.tex index 82093c8..1a0330f 100644 --- a/Analisi I/Parte teorica/2023-03-30, Es. applicazione dei teoremi sulla continuità/main.tex +++ b/Analisi I/Parte teorica/2023-03-30, Es. applicazione dei teoremi sulla continuità/main.tex @@ -134,12 +134,12 @@ \[ \tilde f = \system{ f(x) & \text{se } x \neq a, b, \\ \ell_a & \text{se } x = a, \\ \ell_b & \text{se } x = b.} \] - Allora vale che $\tilde f$ è continua in $\overline I$\footnote{Come + Allora\footnote{Come già riscontrato, vale un risultato ancora più forte: data un'estensione $\tilde f$ di $f$ in $\overline I$, $\tilde f$ è continua se e solo se i valori estesi sono esattamente i limiti della funzione nei punti di $I \setminus \overline I$; e quindi - l'estensione continua è ben definita, e unica del suo genere.}. + l'estensione continua è ben definita, e unica del suo genere.} vale che $\tilde f$ è continua in $\overline I$. \end{exercise} \begin{solution} @@ -160,14 +160,32 @@ %TODO: dimostrare che se limite sinistro e destro coincidono, allora esiste il limite ed è lo stesso del limite sinistro e destro. \begin{exercise} - Si trovi un esempio di $f : X \to \RRbar$, dove, dato $\xbar$ punto + Si trovi un esempio di funzione $f : X \to \RRbar$, dove, dato $\xbar$ punto di accumulazione di $X$, $f(x) \tendsto{\xbar} \ell$, ma - $\exists (x_n) \subseteq X$ tale che $x_n \tendston \xbar$, ma + $\exists \, (x_n) \subseteq X$ tale che $x_n \tendston \xbar$, ma $f(x_n)$ non tende a $\ell$ per $n \to \infty$. \end{exercise} \begin{solution} + Sia $f : \RR \to \RR$ tale che: + \[ f(x) = \system{0 & \text{se } x = 0, \\ 1 & \text{altrimenti}.}\] + + Si consideri allora la successione $(x_n) \subseteq X$ tale che: + + \[ x_n = \system{ 0 & \text{se } n \text{ è pari}, \\ \frac{1}{n} & \text{altrimenti}. } \] + + Si mostra che $x_n \tendston 0$. Infatti, sia $I = [-\eps, \eps]$, con $\eps > 0$, un intorno di $0$. + Allora per $n > \frac{1}{\eps}$ vale che $x_n \in I$ (infatti $0$ vi appartiene sempre, e $0 < \frac{1}{n} < \eps$); + da cui si ricava proprio che $x_n \tendston 0$. \\ + + Chiaramente $f(x) \tendsto{0} 1$. È sufficiente mostrare allora che $f(x_n)$ non tende a $1$ per + $n \to \infty$. Si consideri la sottosuccessione $f(x_{2n})$: poiché $f(x_{2n}) = f(0) = 0$, la + sottosuccessione presa in considerazione è costante, e quindi $f(x_{2n}) \tendston 0$. + Anche la sottosuccessione $f(x_{2n + 1})$ è costante, e vale che $f(x_{2n + 1}) = f(\frac{1}{n}) = 1$, + e quindi $f(x_{2n+1}) \tendston 1$. Poiché allora il limite di $f(x_n)$, se esistesse, dovrebbe essere + uguale a quello di ambo le sottosuccessioni considerate, ed il limite è unico, $f(x_n)$ non ammette + limite, proprio come volevasi dimostrare. \end{solution} \begin{exercise} @@ -186,7 +204,7 @@ da $\xbar$, assurdo dal momento che $I(\xbar)$ non ne contiene uno per costruzione; ma $x$ non può neanche appartenere a $X \setminus J$, dacché in tal modo si può sempre costruire con errore a piacimento - un intorno più piccolo di $I(x)$ tale che sia disgiunto con $J$, + un intorno più piccolo di $J$ tale che sia disgiunto con $I(x)$, \Lightning. Dal momento che $\QQ$ è denso in $\RRbar$, si può allora sempre associare a $J(\xbar)$ un numero razionale $q$ al suo interno. In questo modo si può costruire una funzione $f : X \to \QQ$,